\documentclass[a4paper,12pt]{article} \usepackage[utf8]{inputenc} \usepackage[english,vietnamese]{babel} \usepackage{amsmath} \usepackage{amssymb} \usepackage{enumerate} \usepackage{mathtools} \usepackage{multicol} \usepackage{pgfplots} \usepackage{siunitx} \usetikzlibrary{shapes.geometric,angles,quotes} \newcommand{\ud}{\,\mathrm{d}} \newcommand{\curl}{\mathrm{curl}} \newcommand{\del}{\mathrm{div}} \newcommand{\unit}[1]{\hat{\textbf #1}} \newcommand{\anonym}[2]{\left(#1 \mapsto #2\right)} \newcommand{\tho}[3][]{\dfrac{\partial #1 #2}{\partial #3 #1}} \newcommand{\leibniz}[3][]{\dfrac{\mathrm{d} #1 #2}{\mathrm{d} #3 #1}} \newcommand{\chain}[3]{\tho{#1}{#2}\tho{#2}{#3}} \newcommand{\exercise}[1]{\noindent\textbf{#1.}} \title{Cuculutu Review} \author{Nguyễn Gia Phong} \date{Summer 2019} \begin{document} \maketitle \setcounter{section}{13} \section{Partial Derivatives} \setcounter{subsection}{1} \subsection{Limits et Continuity} \exercise{37} Determine the set of points at which the function is continuous. \[f(x, y) = \begin{dcases} \frac{x^2 y^3}{2x^2 + y^2} &\text{if }(x, y) \neq (0, 0)\\ 1 &\text{if }(x, y) = (0, 0) \end{dcases}\] By AM-GM inequality, \[x^2 + x^2 + y^2 \geq 3x^2|y| \iff \frac{x^2|y^3|}{3x^2|y|} \geq \frac{x^2|y^3|}{2x^2 + y^2} \geq 0 \iff \frac{-y^2}{3} \leq \frac{x^2 y^3}{2x^2 + y^2} \leq \frac{y^2}{3}\] Since $\pm y^2/3 \to 0$ as $y \to 0$, by the Squeeze Theorem, \[\lim_{x\to 0\atop y\to 0}f(x, y) = 0 \neq f(0, 0)\] Therefore $f$ is discontiuous at (0, 0). On $\mathbb R^2\backslash\{0\}$, $f$ is a rational function and thus is continuous on its domain. \exercise{44} Let \[f(x, y) = \begin{dcases} 0 &\text{if }y \leq 0\text{ or }y \geq x^4\\ 1 &\text{if }0 < y < x^4 \end{dcases}\] \begin{enumerate}[(a)] \item For all paths of the form $y = mx^a$ with $a < 4 \iff 4 - a > 0$, consider the function $g(x) = |y| - x^4 = |m|\cdot|x|^a - |x|^4$: \[g(x) \geq 0 \iff |m|\cdot|x|^a \geq |x|^4 \iff |x| \leq \sqrt[4-a]{|m|}\] When this condition is met, either $y \leq 0$ or $y = |y| \geq x^4$, so $f(x, y) = 0$. Therefore $f(x, y) = 0 \to 0$ as $(x, y) \to (0, 0)$ on \[\left\{(x, y)\,\Big|\, x \in \left[-\sqrt[4-a]{|m|}, \sqrt[4-a]{|m|}\right] \cap D\right\}\] which includes the point (0, 0) if the domain $D$ of $x \mapsto mx^a$ does. \item It is trivial that $f(0, 0) = 0$. Along $y = x^4/2$, for $x \neq 0$, \[x^4 - y = x^4 - \frac{x^4}{2} = \frac{x^4}{2} > 0 \iff y < x^4 \Longrightarrow f(x, y) = 1\] Hence \[\lim_{x\to 0\atop y\to 0} f\left(x, \frac{x^4}{2}\right) = 1 \neq f(0, 0) = 0\] or $f$ is discontiuous on $y = x^4/2$ at (0, 0). \item Using the same reasoning, one may also easily show that $f$ is discontiuous on the entire curve $y = x^4/20$. \end{enumerate} \subsection{Partial Derivatives} \exercise{33} Find the first partial derivatives of the function. \begin{align*} w &= \ln(x + 2y + 3z)\\ \tho{w}{x} &= \frac{1}{x + 2y + 3z}\cdot\tho{(x + 2y + 3z)}{x} = \frac{1}{x + 2y + 3z}\\ \tho{w}{y} &= \frac{1}{x + 2y + 3z}\cdot\tho{(x + 2y + 3z)}{y} = \frac{2}{x + 2y + 3z}\\ \tho{w}{z} &= \frac{1}{x + 2y + 3z}\cdot\tho{(x + 2y + 3z)}{z} = \frac{3}{x + 2y + 3z} \end{align*} \exercise{50} Use implicit differentiation to find $\partial z/\partial x$ and $\partial z/\partial y$. \[yz + x\ln y = z^2 \Longrightarrow \begin{dcases} y\tho{z}{x} + \ln y &= 2z\tho{z}{x}\\ z + \frac{x}{y} &= 2z\tho{z}{y}\\ \end{dcases} \iff \begin{dcases} \frac{\ln y}{2z - y} &= \tho{z}{x}\\ 2 + \frac{x}{2yz} &= \tho{z}{y} \end{dcases}\] \exercise{66} Find $g_{rst}$. \begin{multline*} g(r, s, t) = e^r\sin(st) \Longrightarrow g_r = e^r\sin(st)\\ \Longrightarrow g_{rs} = se^r\cos(st) \Longrightarrow g_{rst} = -ste^r\sin(st) \end{multline*} \exercise{101} Let \[f(x, y) = \begin{dcases} \frac{x^3y + xy^3}{x^2 + y^2} &\text{if } (x, y) \neq (0, 0)\\ 0 &\text{if } (x, y) = (0, 0) \end{dcases}\] \begin{enumerate}[(a)] \item Graph $f$. \begin{tikzpicture} \begin{axis}[xlabel={x}, ylabel={y}] \addplot3[surf]{(x^3 * y - x * y^3) / (x^2 + y^2)}; \end{axis} \end{tikzpicture} \item Find the first partial derivatives of $f$ when $(x, y) \neq (0, 0)$. \begin{align*} \tho{f}{x} &= \frac{(x^2 + y^2)\tho{(x^3y - xy^3)}{x} - (x^3y - xy^3)\tho{(x^2 + y^2)}{x}}{(x^2 + y^2)^2}\\ &= \frac{(x^2 + y^2)(3x^2y - y^3) - 2x(x^3y - xy^3)}{(x^2 + y^2)^2}\\ &= \frac{x^4y + 4x^2y^3 - y^5}{x^4 + 2x^2y^2 + y^4} \end{align*} \begin{align*} \tho{f}{x} &= \frac{(x^2 + y^2)(x^3 - 3xy^2) - 2y(x^3y - xy^3)}{(x^2 + y^2)^2}\\ &= \frac{x^5 - 4x^3y^2 - xy^4}{x^4 + 2x^2y^2 + y^4} \end{align*} \item Find $f_x$, $f_y$ at (0, 0). \begin{align*} f_x(0, 0) &= \lim_{h\to 0}\frac{f(h, 0) - f(0, 0)}{h} = \lim_{h\to 0}\frac{\frac{h^30 - h0^3}{h^2 + 0^2} - 0}{h} = \lim_{h\to 0}0 = 0\\ f_y(0, 0) &= \lim_{h\to 0}\frac{f(0, h) - f(0, 0)}{h} = \lim_{h\to 0}\frac{0 - 0}{h} = \lim_{h\to 0}0 = 0 \end{align*} \item Show that $f_{xy}(0, 0) = -1$ and $f_{yx}(0, 0) = 1$. \begin{align*} f_{xy}(0, 0) &= \lim_{h\to 0}\frac{f_x(0, h) - f_x(0, 0)}{h} = \lim_{h\to 0}\frac{\frac{0 + 0 - h^5}{0 + 0 + h^4} - 0}{h} = \lim_{h\to 0}-1 = -1\\ f_{yx}(0, 0) &= \lim_{h\to 0}\frac{f_y(h, 0) - f_y(0, 0)}{h} = \lim_{h\to 0}\frac{\frac{h^5 + 0 + 0}{h^4 + 0 + 0} - 0}{h} = \lim_{h\to 0}1 = 1 \end{align*} \item The result of part (d) does not contradict Clairaut's Theorem, which only covers the case $f_{xy}$ and $f_{yx}$ are continuous at (0, 0). Using GeoGebra we get the second derivatives of $f$ on $\mathbb R\backslash\{0\}$ as followed: \[f_{xy} = f_{yx} = \frac{x^6 + 9x^4y^2 - 9x^2y^4 - y^6}{(x^2 + y^2)^3}\] Since $f_{xy}(x, 0) = x^6/x^6 \to 1$ while $f_{xy} = -y^6/y^6\to -1$ as $(x, y) \to (0, 0)$ the second derivative is discontinuous at origin. \end{enumerate} \setcounter{subsection}{5} \subsection{Directional Derivatives} \exercise{17} Find the directional derivative of $h(r,s,t) = \ln(3r + 6s + 9t)$ at (1, 1, 1) in the direction of $\mathbf v = 4\unit\i + 12\unit\j + 6\unit k$. From gradient of $h$ \[\nabla h = \frac{3\unit\i + 6\unit\j + 9\unit k}{3r + 6s + 9t} \Longrightarrow \nabla h(1, 1, 1) = \frac{\unit\i}{6} + \frac{\unit\j}{3} + \frac{\unit k}{2}\] and unit vector of $\mathbf v$ \[\unit v = \frac{2\unit\i}{7} + \frac{6\unit\j}{7} + \frac{3\unit k}{7}\] we can compute the direction derivative as \[\mathrm D_{\unit v}(1, 1, 1) = \nabla h(1, 1, 1)\cdot\unit v = \frac{1}{21} + \frac{4}{7} + \frac{3}{14} = \frac{23}{42}\] \subsection{Maximum and Minimum Values} \exercise{18} Find the local maximum and minimum values and saddle point(s) of the function. If you have three-dimensional graphing software, graph the function with a domain and viewpoint that reveal all the important aspects of the function. \[f(x, y) = \sin x\sin y,\qquad -\pi < x < \pi,\qquad -\pi < y < \pi\] \begin{multicols}{2} \begin{align*} &\Longrightarrow\begin{cases} f_x = \cos x\sin y\\ f_y = \sin x\cos y \end{cases}\\ &\Longrightarrow\begin{cases} f_{xx} = f_{yy} = -\sin x\sin y\\ f_{xy} = f_{yx} = \sin x\sin y \end{cases}\\ &\Longrightarrow D = f_{xx}f_{yy} - f_{xy}^2 = 0 \end{align*} For $f_x = f_y = 0$, either $x = y = 0$ or $x, y \in \{\pm\pi/2\}$. $D$ does not indicate if $f$ has local extreme values at these critical points. \noindent\begin{tikzpicture}[domain=-pi:pi] \begin{axis}[xlabel={x}, ylabel={y}] \addplot3[surf]{sin(deg(x)) * sin(deg(y))}; \end{axis} \end{tikzpicture} \end{multicols} It is clear that $f$ has 2 local maximums of 1 at $x = y = \pm\pi$ and 2 local minimum of -1 at $x = -y = \pm\pi$, since these are its absolute extreme values as well. Suppose $f(0, 0)$ is a local minimum. Then, by definition, $f(a, b) \geq f(0, 0) = 0$ if $(a, b)$ is sufficiently close to origin (say, at most within $[-\pi/2, \pi/2]^2$). However, for all $a$, $b$ satisfying $ab < 0$, $f(a, b) = \sin a\sin b < 0$, thus our assumption is incorrect. Similarly, $f$ does not has a local maximum at origin because \[\forall a, b \in \left[-\frac{\pi}{2}, \frac{\pi}{2}\right]: ab > 0, \qquad f(a, b) = \sin a\sin b > 0 = f(0, 0)\] Therefore (0, 0) is a saddle point. \exercise{35} Find the absolute extreme values of $f(x, y) = 2x^3 + y^4$ on the unit disc. The critical points of $f$ occur when \[f_x = f_y = 0 \iff 6x^2 = 4y^3 = 0 \iff x = y = 0\] at which $f(x, y) = f(0, 0) = 0$. On the unit circle, as $y^2 = 1 - x^2$, let \[g(x) = f(x, y) = 2x^3 + (1 - x^2)^2 = x^4 + 2x^3 - 2x^2 + 1\] Within $[-1, 1]$, $g'(x) = 4x^3 + 6x^2 - 4x = 0$ if and only if $x = 0$ or $x = 0.5$. Since $g(-1) = -2$, $g(0) = 1$, $g(0.5) = 0.8125$ and $g(1) = 2$, the absolute minimum and maximum of $g$ on $[-1, 1]$ are respectively $g(-1) = -2$ and $g(1) = 2$. Thus on the boundary, the minimum value of $f$ is -2 at $(-1, \pm 1)$ and the maximum value is 2 at $(1, \pm 1)$. \exercise{46} Find the dimensions of the box with volume $1000\text{ cm}^3$ that has minimal surface area. Let the dimensions of the box be $x, y, z$ in dm, $x, y, z$ are positive and $xyz = 1$. Total surface area of the box would then be \[S(x, y, z) = 2xy + 2yz + 2zx\] By AM-GM inequality, \[S(x, y, z) \geq 2\cdot 3\sqrt{xy\cdot yz\cdot zx} = 6\] Thus $S$ has its absolute minumum of 6 at $x = y = z = 1$. \exercise{53} If the length of the diagonal of a rectangular box must be $L$, what is the largest possible volume? Let the dimensions of the box be three positive numbers $x, y, z$, $x^2 + y^2 + z^2 = L^2$. The volume of the box would then be $V(x, y, z) = xyz$. By AM-GM inequality, \[V(x, y, z) = \sqrt{x^2 y^2 z^2} \leq \frac{x^2 + y^2 + z^2}{3} = \frac{L^2}{3}\] Thus $V$ has its absolute maximum of $L^2/3$ at $x = y = z = L/\sqrt 3$. \subsection{Lagrange Multipliers} \exercise{12} Use Lagrange multipliers to find the maximum and minimum values of $f(x, y, z) = x^4 + y^4 + z^4$ subject to $g(x, y, z) = x^2 + y^2 + z^2 = 1$. Extreme values of $f$ occur when \[\begin{cases} \nabla f = \lambda\nabla g\\ g(x, y, z) = 1 \end{cases} \iff\begin{cases} \langle 4x^3, 4y^3, 4z^3\rangle = \lambda\langle 2x, 2y, 2z\rangle \neq \mathbf 0\\ x^2 + y^2 + z^2 = 1 \end{cases}\] \begin{enumerate} \item For $\lambda = 2/3$, $x^2 = y^2 = z^2 = 1/3 = f(x, y, z)$. \item For $\lambda = 1$ and $(x^2, y^2, z^2) \in \{(0, 1/2, 1/2), (1/2, 0, 1/2), (1/2, 1/2, 0)\}$, $f(x, y, z) = 1/2$. \item For $\lambda = 2$ and $(x^2, y^2, z^2) \in \{(1, 0, 0), (0, 1, 0), (0, 0, 1)\}$, $f(x, y, z) = 1$. \end{enumerate} Therefore, subject to the given constrain, $f$ has absolute maximum of 1 and minimum of 1/3.\pagebreak \exercise{42} Find the maximum and minimum volumes of a rectangular box whose surface area is $1500\text{ cm}^2$ and whose total edge length is 200 cm. Let the dimensions of the box be $x, y, z$ in dm, with $x, y, z$ are positive, $2xy + 2yz + 2zx = 15$ and $4x + 4y + 4z = 20$. From these constrains, we can easily obtain $x + y = 5 - z$ and \[xy + (x + y)z = \frac{15}{2} \iff xy = \frac{15}{2} - 5z + z^2\] Thus with $0 < z < 5$ the volume of the box is \[V = xyz = z^3 - 5z^2 + \frac{15z}{2}\] whose critical points are \[\leibniz{V}{z} = 3z^2 - 10z + \frac{15}{2} = 0 \iff z = \frac{10 \pm \sqrt{10}}{6}\] at which $V = \dfrac{175 \pm 5\sqrt{10}}{54}$. On the other hand, the constrains are equivalent to \[\begin{cases} x^2 + y^2 + z^2 = 10\\ x + y + z = 5 \end{cases}\] or the intersection of a sphere and a plane, which result in a circle $C$. Hence the range of $z$ would be between $a$ and $b$, whereas each of $z = a$ and $z = b$ only has one point in common with $C$. Since all surfaces $x^2 + y^2 + z^2 = 10$, $x + y + z = 5$, $z = a$ and $z = b$ has $x = y$ as their plane of symmetry, these two points must be on $x = y$ as well: \begin{align*} \begin{cases} 2x^2 + z^2 = 10\\ 2x + z = 5 \end{cases} \iff&\begin{cases} 2x^2 + (5 - 2x)^2 = 10\\ z = 5 - 2x \end{cases}\\ \iff&\begin{cases} 6x^2 - 20x + 15 = 0\\ z = 5 - 2x \end{cases}\\ \iff&\begin{dcases} x = \frac{10 \pm \sqrt{10}}{6}\\ z = \frac{5 \pm \sqrt{10}}{3} \end{dcases}\\ \Longrightarrow &\,V = \dfrac{175 \pm 5\sqrt{10}}{54} \end{align*} These are the maximum and minimum volumes of the given box. \section{Multiple Integrals} \setcounter{subsection}{1} \subsection{Interated Integrals} \exercise{19} Calculate the double integral. \begin{align*} \int_0^{\pi/6}\int_0^{\pi/3}x\sin(x + y)\ud y\ud x &= \int_0^{\pi/6}\left[-x\cos(x + y)\right]_{y=0}^{y=\pi/3}\ud x\\ &= \int_0^{\pi/6}x\left(\cos x - \cos\left(x + \frac\pi 3\right)\right)\ud x\\ &= \int_0^{\pi/6}x\cos\left(x - \frac\pi 3\right)\ud x\\ &= \int_0^{\pi/6}x\ud\cos\left(x - \frac\pi 3\right)\\ &= \left[x\sin\left(x - \frac\pi 3\right)\right]_0^{\pi/6} - \int_0^{\pi/6}\sin\left(x - \frac\pi 3\right)\ud x\\ &= -\frac{\pi}{12} + \left[\cos\left(x - \frac\pi 3\right)\right]_0^{\pi/6}\\ &= \frac{\sqrt 3}{2} - \frac{1}{2} - \frac{\pi}{12} \end{align*} \exercise{28} Find the volume of the solid enclosed by the surface $z = 1 + e^x\sin y$ and the planes $x = \pm 1$, $y = 0$, $y = \pi$ and $z = 0$. \begin{align*} \int_0^\pi\int_{-1}^1(1 + e^x\sin y)\ud x \ud y &= \int_0^\pi\left[x + e^x\sin y\right]_{x=-1}^{x=1}\ud y\\ &= \int_0^\pi\left(2 + \left(e - \frac{1}{e}\right)\sin y\right)\ud y\\ &= \left[2x + \left(\frac{1}{e} - e\right)\cos y\right]_0^\pi\\ &= 2\pi \end{align*} \subsection{Double Integrals over General Regions} \exercise{10} Evaluate the double integral. \begin{align*} \int_1^e\int_0^{\ln x}x^3\ud y\ud x &= \int_1^e x^3\ln x\ud x\\ &= \int_1^e\ln x\ud\frac{x^4}{4}\\ &= \left.\frac{x^4\ln x}{4}\right]_1^e - \int_1^e\frac{x^4}{4}\ud\ln x\\ &= e^4 - \int_1^e\frac{x^3}{4}\ud x\\ &= e^4 - \left.\frac{x^4}{16}\right]_1^e\\ &= \frac{15e^4 + 1}{16} \end{align*} \exercise{16} Set up iterated integrals for both orders of integration. Then evaluate the double integral using the easier order and explain why it’s easier. \begin{multline*} I = \iint_D y^2 e^{xy}\ud A,\qquad D\text{ is bounded by }y = x, y = 4, x = 0\\ \Longrightarrow I = \int_0^4\int_x^4 y^2 e^{xy}\ud y\ud x = \int_0^4\int_0^y y^2 e^{xy}\ud x\ud y \end{multline*} Since $y^2 e^{xy}$ is simply an exponential function of $x$, it would be easier to evaluate \begin{align*} I &= \int_0^4\int_0^y y^2 e^{xy}\ud x\ud y\\ &= \int_0^4\left[y^3 e^{xy}\right]_{x=0}^{x=y}\ud y\\ &= \int_0^4 y^3 e^{y^2}\ud y = \int_0^4 y^2\ud\frac{e^{y^2}}{2}\\ &= \left.\frac{y^2 e^{y^2}}{2}\right]_0^4 - \int_0^4\frac{e^{y^2}}{2}\ud y^2\\ &= 8e^{16} - \int_0^{16}\frac{e^z}{2}\ud z\\ &= 8e^{16} - \left.\frac{e^z}{2}\right]_0^{16} = \frac{15e^{16}}{2} \end{align*} \exercise{31} Find the volume of the solid bounded by the cylinder $x^2 + y^2 = 1$ and the plane $y = z$ in the first octant. \[\int_0^1\int_0^{\sqrt{1-x^2}}y\ud y\ud x = \int_0^1\frac{1 - x^2}{2}\ud x = \frac{1}{3}\] \subsection{Double Integrals in Polar Coordinates} \exercise{13} Evaluate the given integral by changing to polar coordinates. \[I = \iint_R\arctan\frac{y}{x}\ud A,\qquad \text{where }R = \{(x, y)\,|\,1 \leq x^2 + y^2 \leq 4, 0 \leq y \leq x\}\] In polar coordinates, \[R = [1, 2]\times \left[0, \frac\pi 4\right]\] thus \begin{align*} I &= \int_0^{\pi/4}\int_1^2\arctan\frac{r\sin\theta}{r\cos\theta} r\ud r\ud\theta\\ &= \int_0^{\pi/4}\int_1^2\arctan\tan\theta r\ud r\ud\theta\\ &= \int_0^{\pi/4}\int_1^2\theta r\ud r\ud\theta\\ &= \int_0^{\pi/4}\frac{3\theta}{2}\ud r\ud\theta\\ &= \frac{3\pi^2}{64} \end{align*} \begin{multicols*}{2} \noindent\begin{tikzpicture}[domain=-pi:pi] \begin{axis}[legend pos=south east, xlabel={$\theta$}, ylabel={$r$}, axis x line = middle, axis y line = middle, enlarge y limits={rel=0.1}, enlarge x limits={rel=0.1}] \addplot[color=magenta]{1}; \addplot[color=green]{2 * cos(deg(x))}; \legend{$r = 1$, $r = 2\cos\theta$} \end{axis} \end{tikzpicture} \exercise{17} Use a double integral to find the area of the region inside $C_1: (x - 1)^2 + y^2 = 1$ and outside $C_0: x^2 + y^2 = 1$.\\ In polar coordinates $C_1$ has the equation $r = 2\cos\theta$ and the equation of $C_0$ is $r = 1$. Therefore the given region is within $1 \leq r \leq 2\cos\theta$, whereas $\theta \in [-\pi, \pi]$. \end{multicols*} Since on $[-\pi, \pi]$, $2\cos\theta \geq 1 \iff -\pi/3 \leq \theta \leq \pi/3$, the area of the given region is \begin{align*} \int_{-\pi/3}^{\pi/3}\int_1^{2\cos\theta}r\ud r\ud\theta &= \int_{-\pi/3}^{\pi/3}\frac{4\cos^2\theta - 1}{2}\ud\theta\\ &= \int_{-\pi/3}^{\pi/3}\left(2\cos^2\theta - 1 + \frac{1}{2}\right)\ud\theta\\ &= \int_{-\pi/3}^{\pi/3}\left(\cos 2\theta + \frac{1}{2}\right)\ud\theta\\ &= \left[\frac{\sin 2\theta + \theta}{2}\right]_{-\pi/3}^{\pi/3}\\ &= \frac{\sqrt 3}{2} + \frac\pi 3 \end{align*} \subsection{Applications of Double Integrals} \exercise{5} Find the mass and center of mass of the lamina that occupies the region triangular $D$ with vertices (0, 0), (2, 1), (0, 3) and has the given density function $\rho(x, y) = x + y$. \begin{align*} m &= \iint_D(x + y)\ud A\\ &= \int_0^2\int_{x/2}^{3-x}(x+y)\ud y\ud x\\ &= \int_0^2\frac{36 - 9x^2}{8}\ud x\\ &= 9 - 3 = 6 \end{align*} \begin{align*} \bar x &= \iint_D\frac{x(x + y)}{m}\ud A& \bar y &= \iint_D\frac{y(x + y)}{m}\ud A\\ &= \int_0^2\int_{x/2}^{3-x}\frac{x^2 + xy}{6}\ud y\ud x& &= \int_0^2\int_{x/2}^{3-x}\frac{xy + y^2}{6}\ud y\ud x\\ &= \int_0^2\frac{12x - 3x^3}{16}\ud x& &= \int_0^2\frac{6 - 3x}{4}\ud x\\ &= \frac{3}{4}& &= \frac{3}{2} \end{align*} \subsection{Surface Area} \exercise{7} Find the area of the part of the hyperbolic paraboloid $z = y^2 - x^2$ that lies between the cylinders $x^2 + y^2 = 1$ and $x^2 + y^2 = 4$. \begin{align*} &\iint_D\sqrt{1 + \left(\tho{(y^2 - x^2)}{x}\right)^2 + \left(\tho{(y^2 - x^2)}{y}\right)^2}\ud A\\ = &\iint_D\sqrt{1 + 4x^2 + 4y^2}\ud A\\ = &\int_0^{2\pi}\int_1^2 r\sqrt{1 + 4r^2\cos^2\theta + 4r^2\sin^2\theta} \ud r\ud\theta\\ = &\int_1^2\pi\sqrt{1 + 4r^2}\ud r^2\\ = &\int_1^4\pi\sqrt{1 + 4t}\ud t\\ = &\,\pi\left[\frac{(1 + 4t)^{1.5}}{6}\right]_1^4\\ = &\,\frac{17^{1.5} - 5^{1.5}}{6}\pi \end{align*} \section{Vector Calculus} \setcounter{subsection}{1} \subsection{Line Integrals} \exercise{12} Evaluate the integral, where $C$ is the given curve. \[I = \int_C(x^2 + y^2 + z^2)\ud s,\qquad C: x = t, y = \cos 2t, z = \sin 2t, 0 \leq t \leq 2\pi\] \begin{align*} I &= \int_0^{2\pi}(x^2 + y^2 + z^2)\sqrt{\left(\leibniz{x}{t}\right)^2 + \left(\leibniz{z}{t}\right)^2 + \left(\leibniz{z}{t}\right)^2}\ud t\\ &= \int_0^{2\pi}(t^2 + \cos^2 2t + \sin^2 2t) \sqrt{\left(\leibniz{t}{t}\right)^2 + \left(\leibniz{\cos 2t}{t}\right)^2 + \left(\leibniz{\sin 2t}{t}\right)^2}\ud t\\ &= \int_0^{2\pi}(t^2 + 1)\sqrt 2\ud t = \frac{8\pi\sqrt 2}{3} + 2\pi\sqrt 2 \end{align*} \exercise{15} With $C$ is the line segment from (1, 0, 0) to (4, 1, 2), $x = 3t + 1$, $y = t$, $z = 2t$, whereas $0 \leq t \leq 1$ and \begin{align*} J &= \int_C z^2\ud x + x^2\ud y + y^2\ud z\\ &= \int_0^1 z^2\leibniz{x}{t}\ud t + x^2\leibniz{y}{t}\ud t + y^2\leibniz{z}{t}\ud t\\ &= \int_0^1(x^2 + 2y^2 + 3z^2)\ud t\\ &= \int_0^1(9t^2 + 6t + 1 + 2t^2 + 12t^2)\ud t\\ &= \int_0^1(23t^2 + 6t + 1)\ud t\\ &= \left[\frac{23t^3}{3} + 3t^2 + t\right]_0^1 = \frac{35}{3} \end{align*} \exercise{39} Find the work done by the force field $\mathbf F(x, y) = \langle x, y + 2\rangle$ is moving an object along an arch of the cycloid $\mathbf r(t) = \langle t-\sin t, 1-\cos t\rangle$, $0 \leq t \leq 2\pi$. \begin{align*} W &= \int_C\mathbf F\cdot\ud\mathbf r\\ &= \int_0^{2\pi}\mathbf F\cdot\leibniz{\mathbf r}{t}\ud t\\ &= \int_0^{2\pi}\langle x, y+2\rangle\cdot \left<\leibniz{x}{t}, \leibniz{y}{t}\right>\ud t\\ &= \int_0^{2\pi}\langle t-\sin t, 3-\cos t\rangle\cdot \left<1-\cos t, \sin t\right>\ud t\\ &= \int_0^{2\pi}\left(t - t\cos t + 2\sin t\right)\ud t\\ &= \left[\frac{t^2}{2} - t\sin t - 3\cos t\right]_0^{2\pi} = 2\pi^2 \end{align*} \subsection{The Fundamental Theorem for Line Integral} \exercise{19} Show that the line integral is independent from any path $C$ from (1, 0) to (2, 1) and evaluate the integral. \[\int_C\frac{2x}{e^y}\ud x + \left(2y - \frac{x^2}{e^y}\right)\ud y = \int_C\left(\frac{2x}{e^y}\unit\i + 2y\unit\j - \frac{x^2}{e^y}\unit\j\right) \cdot\ud(x\unit\i + y\unit\j)\] Since on $\mathbb R^2$ \[\tho{}{y}\frac{2x}{e^y} = \frac{-2x}{e^y} = \tho{}{x}\left(2y - \frac{x^2}{e^y}\right)\] the function \[\mathbf F(x, y) = \frac{2x}{e^y}\unit\i + 2y\unit\j - \frac{x^2}{e^y}\unit\j\] is conservative and thus the given line integral is independent from path. Let $f$ be a differentiable of $(x, y)$ that $\nabla f = \mathbf F$. One function satisfying this is \[f(x, y) = y^2 + \frac{x^2}{e^y}\] By the fundamental theorem for line integrals, \[\int_C\mathbf F\cdot\ud\mathbf r = f(2, 1) - f(1, 0) = \frac{4}{e}\] \subsection{Green's Theorem} \exercise{6} Use Green’s Theorem to evaluate the line integral along the given positively oriented rectangle with vertices (0, 0), (5, 0), (5, 2) and (0, 2). \begin{align*} \int_C\cos y\ud x + x^2\sin y\ud y &= \int_0^5\int_0^2\left(\tho{x^2\sin y}{x} - \tho{\cos y}{y}\right)\ud y\ud x\\ &= \int_0^5\int_0^2(2x\sin y + \sin y)\ud y\ud x\\ &= \int_0^5(2x + 1)(1 - \cos 2)\ud x\\ &= 30 - 30\cos 2 \end{align*} \begin{multicols}{2} \noindent\begin{tikzpicture}[domain=-pi/2:pi/2] \begin{axis}[legend pos=north east, xlabel={$x$}, ylabel={$y$}, axis x line = middle, axis y line = middle, enlarge y limits={rel=0.1}, enlarge x limits={rel=0.1}] \addplot[->,>=stealth,color=blue]{cos(deg(x))}; \addplot[<-,>=stealth,color=orange]{0}; \legend{$r = \cos x$, $r = 0$} \end{axis} \end{tikzpicture} \exercise{12} Use Green's Theorem to evaluate the line integral along the path $C$ including the curve $y = \cos x$ from $(-\pi/2, 0)$ to $(\pi/2, 0)$ and the line segment connecting these two points. Since the curve is negatively oriented, by Green's Theorem, \end{multicols} \begin{align*} &\int_C(e^{-x} + y^2)\ud x + (e^{-y} + x^2)\ud y\\ = &\,-\int_{-\pi/2}^{\pi/2}\int_0^{\cos x}\left( \tho{}{x}(e^{-y} + x^2) - \tho{}{y}(e^{-x} + y^2)\right)\ud y\ud x\\ = &\int_{\pi/2}^{-\pi/2}\int_0^{\cos x}(2x - 2y)\ud y\ud x\\ = &\int_{\pi/2}^{-\pi/2}(2x\cos x - \cos^2 x)\ud x\\ = &\,\frac 1 2\int_{-\pi/2}^{\pi/2}(\cos 2x + 1)\ud x - \int_{-\pi/2}^{\pi/2}2x\ud\sin x\\ = &\left[\frac{\sin 2x}{4} + \frac{x}{2} - 2x\sin x - 2\cos x\right]_{-\pi/2}^{\pi/2} = \frac\pi 2 \end{align*} \subsection{Curl and Divergence} This section is to aid my revision of Electromagnetism. First, on $\mathbb R^3$, we define \[\nabla = \unit\i\tho{}{x} + \unit\j\tho{}{y} + \unit k\tho{}{z}\] then the curl of vector field $\mathbf F = P\unit\i + Q\unit\j + R\unit k$ is \begin{multline*} \curl\mathbf F = \nabla\times\mathbf F = \begin{vmatrix} \unit\i & \unit\j & \unit k\\ \tho{}{x} & \tho{}{y} & \tho{}{z}\\ P & Q & R \end{vmatrix}\\ = \unit\i\left(\tho{R}{y} - \tho{Q}{z}\right) + \unit\j\left(\tho{P}{z} - \tho{R}{x}\right) + \unit k\left(\tho{Q}{x} - \tho{P}{y}\right) \end{multline*} If $f$ is a function of three variables that has continuous second-order partial derivatives, then $\curl(\nabla f) = \mathbf 0$. On the other hand, if $\curl\mathbf F = \mathbf 0$ then $\mathbf F$ is a conservative vector field (preconditions: $P$, $Q$ and $R$ must be partially differentiable). Similarly, the divergence of vector field $\mathbf F$ is defined as \[\del\mathbf{F} = \nabla\cdot\mathbf F = \unit\i\tho{P}{x} + \unit\j\tho{Q}{y} + \unit k\tho{R}{z}\] Trivially, $\nabla\cdot(\nabla\times\mathbf F) = 0$ because the terms cancel in pairs by Clairaut's Theorem. The cool thing about operators is that they can be weirdly combined, e.g. $\del(\nabla f) = \nabla\cdot\nabla f = \nabla^2 f$ and $\nabla^2 F = \nabla\cdot\nabla\cdot\mathbf F$. Now we are able to write Green's Theorem in the vector form \[\oint_{\partial S}\mathbf F\cdot\ud\mathbf r = \iint_S(\curl\mathbf F)\cdot\unit k\ud A\] whereas $\mathbf r(t) = x(t)\unit\i + y(t)\unit\j$. The outward normal vector to the contour is given by $\mathbf n(t) = \leibniz{y}{t}\unit\i - \leibniz{x}{t}\unit\j$. So we have the second vector form of Green's Theorem. \[\oint_{\partial S}\mathbf F\cdot\unit n\ud s = \iint_S\del\mathbf F\ud A\] \subsection{Parametric Surfaces and Their Areas} \exercise{42} Find the area of the part of the cone $z = \sqrt{x^2 + y^2}$ that lies between the plane $y = x$ and the cylinder $y = x^2$. \begin{align*} &\int_0^1\int_{x^2}^x\sqrt{ 1 + \left(\tho{z}{x}\right)^2 + \left(\tho{z}{y}\right)_2}\ud y\ud x\\ =&\int_0^1\int_{x^2}^x\sqrt 2\ud y\ud x = \int_0^1(x - x^2)\sqrt 2\ud y\ud x\\ =&\,\frac{1}{2} - \frac{1}{3} = \frac{1}{6} \end{align*} \section{Second-Order Differential Equations} \subsection{Homogeneous Linear Equations} \exercise{11} Solve the differential equation. \[2\leibniz{^2 y}{t^2} + 2\leibniz{y}{t} - y = 0\] Since the auxiliary equation $2r^2 + 2x - 1 = 0$ has two real and distinct roots $\dfrac{\pm\sqrt 3 - 1}{2}$, the general solution is \[y = c_1\exp\frac{\sqrt 3 - 1}{2}t + c_2\exp\frac{-\sqrt 3 - 1}{2}t\] \exercise{21} Solve the initial value problem. \[y'' - 6y' + 10y = 0,\qquad y(0) = 2,\qquad y''(0) = 3\] Since the auxiliary equation $r^2 - 6x + 10 = 0$ has two complex roots $3\pm i$, the general solution is \[y = e^{3x}(c_1\cos x + c_2\sin x) \Longrightarrow y' = e^{3x}((3c_1 + c_2)\cos x + (3c_2 - c_1)\sin x)\] As $y(0) = 2$, $c_1 = 2$. Similarly, from $y'(0) = 3$, we can obtain $3c_1 - c_2 = 3 \Longrightarrow c_2 = 3$. Thus the solution of the initial value problem is $y = e^{3x}(3\cos x + 2\sin x)$. \subsection{Nonhomogeneous Linear Equations} Solve the differential equation or initial-value problem using the method of undetermined coefficients. \[y'' - 4y' + 5y = e^{-x}\tag{5}\] The auxiliary equation of $y'' - 4y' + 5y = 0$ is $r^2 - 4r + 5 = 0$ with roots $r = 2\pm i$. Hence the solution to the complementary equation is \[y_c = e^{2x}(c_1\cos x + c_2\sin x)\] Since $G(x) = e^{-x}$ is an exponential function, we seek a particular solution of an exponential function as well: \[y_p = Ae^{-x} \Longrightarrow y_p' = -Ae^{-x} \Longrightarrow y_p'' = Ae^{-x}\] Substituting these into the differential equation, we get \[Ae^{-x} - 4Ae^{-x} + 5Ae^{-x} = e^{-x} \iff A = \frac{1}{10}\] Thus the general solution of the exponential equation is \[y = y_c + y_p = e^{2x}(c_1\cos x + c_2\sin x) + \frac{1}{10e^x}\] \[y'' + y' - 2y = x + \sin 2x,\qquad y(0) = 1,\qquad y'(0) = 0\tag{10}\] The auxiliary equation of $y'' + y' - 2y = 0$ is $r^2 + r - 2 = 0$ with roots $r = -2, 1$. Thus the solution to the complementary equation is \[y_c = c_1 e^x + \frac{c_2}{e^{2x}}\] We seek a particular solution of the form \begin{multline*} y_p = Ax + B + C\cos 2x + D\sin 2x\\ \Longrightarrow y_p' = A - 2C\sin 2x + 2D\cos 2x\\ \Longrightarrow y_p'' = -4C\cos 2x - 4D\sin 2x \end{multline*} Substituting these into the differential equation, we get \begin{multline*} (-4C + 2D - 2C)\cos 2x + (-4D - 2C - 2D)\sin 2x + A - 2B - 2Ax = x + \sin 2x\\ \iff\begin{cases} -4C + 2D - 2C = 0\\ -4D - 2C - 2D = 1\\ A - 2B = 0\\ -2A = 1 \end{cases} \iff\begin{cases} A = -1/2\\ B = -1/4\\ C = -1/20\\ D = -3/20 \end{cases} \end{multline*} Thus the general solution of the exponential equation is \begin{multline*} y = y_c + y_p = c_1 e^x + \frac{c_2}{e^{2x}} - \frac{x}{2} - \frac{1}{4} - \frac{\cos 2x}{20} - \frac{3\sin 2x}{20}\\ \Longrightarrow y' = c_1 e^x - \frac{2c_2}{e^{2x}} - \frac{1}{2} + \frac{\sin 2x}{10} - \frac{3\cos 2x}{10} \end{multline*} Since $y(0) = 1$ and $y'(0) = 0$, \[\begin{dcases} c_1 + c_2 - \frac{1}{4} - \frac{1}{20} = 1\\ c_1 - 2c_2 - \frac{3}{10} = 0 \end{dcases} \iff \begin{dcases} c_1 + c_2 = \frac{13}{10}\\ c_1 - 2c_2 = \frac{3}{10} \end{dcases} \iff \begin{dcases} c_1 = \frac{29}{30}\\ c_2 = \frac{1}{3} \end{dcases}\] Therefore the solution to the initial value problem is \[y = \frac{29e^x}{30} + \frac{1}{3e^{2x}} - \frac{x}{2} - \frac{1}{4} - \frac{\cos 2x}{20} - \frac{3\sin 2x}{20}\] \subsection{Applications} \exercise{3} A spring with a mass of 2 kg has damping constant 14, and a force of 6 N is required to keep the spring stretched 0.5 m beyond its natural length. The spring is stretched 1 m beyond its natural length and then released with zero velocity. Find the position of the mass at any time $t$. By Hooke's law, \[k(0.5) = 6 \iff k = 12\] By Newton's second law of motion, \[2\leibniz{^2 x}{t^2} + 14\leibniz{x}{t} + 12x = 0\] Since the auxiliary equation $2r^2 + 14r + 12 = 0$ has two real and distinct roots $r = -6, -1$, the general solution is \[x = \frac{c_1}{e^t} + \frac{c_2}{e^{6t}} \Longrightarrow \leibniz{x}{t} = \frac{-c_1}{e^t} - \frac{6c_2}{e^{6t}}\] From $x(0) = 1$ and $x'(0) = 0$ we get \[\begin{cases} c_1 + c_2 = 1\\ -c_1 - 6c_2 = 0 \end{cases} \iff\begin{cases} c_1 = 6/5\\ c_2 = -1/5 \end{cases}\] Therefore the position at any time $t$ is \[x = \frac{6}{5e^t} - \frac{c_2}{5e^{6t}}\] \setcounter{section}{8} \section{First-Order Differential Equations} \setcounter{subsection}{2} \subsection{Separable Equations} \exercise{8} Solve the differential equation. \begin{align*} \leibniz{y}{\theta} &= \frac{e^y\sin^2\theta}{y\sec\theta}\\ \iff \int\frac{y}{e^y}\ud y &= \int\sin\theta\cos\theta\ud\theta\\ \iff \int-y\ud e^{-y} &= \int\sin^2\theta\ud\sin\theta\\ \iff \int e^{-y}\ud y - \frac{y}{e^y} &= \frac{\sin^3\theta}{3}\\ \iff \frac{1 + y}{e^y} &= C - \frac{\sin^3\theta}{3} \end{align*} \setcounter{subsection}{4} \subsection{Linear Equations} \exercise{28} In a damped RL circuit, the generator supplies a voltage of $E(t) = 40\sin 60t$ volts, the inductance is 1 H, the resistance is 10 $\Omega$ and $I(0) = 1$ A. \begin{align*} &E - L\leibniz{I}{t} - RI = 0\\ \iff &\frac{40}{L}\sin 60t = \leibniz{I}{t} + \frac{RI}{L}\\ \iff &\frac{40e^{tR/L}}{L}\sin 60t = \frac{RI}{L}e^{tR/L} + \leibniz{I}{t}e^{tR/L}\\ \iff &\frac{40}{L}\int e^{tR/L}\sin 60t\ud t = \int\ud Ie^{tR/L}\tag{$*$} \end{align*} Let $J = \int e^{tR/L}\sin 60t\ud t$, \begin{align*} J &= \frac{-1}{60}\int e^{tR/L}\ud\cos 60t\\ &= \frac{1}{60}\int\cos 60t\ud e^{tR/L} - \frac{e^{tR/L}\cos 60t}{60}\\ &= \frac{R}{3600L}\int e^{tR/L}\ud\sin 60t - \frac{e^{tR/L}\cos 60t}{60}\\ &= \frac{R}{3600L}e^{tR/L}\sin 60t - \frac{R}{3600L}\int\sin 60t\ud e^{tR/L} - \frac{e^{tR/L}\cos 60t}{60}\\ &= \frac{R}{3600L}e^{tR/L}\sin 60t - \frac{R^2}{3600L^2}J - \frac{e^{tR/L}\cos 60t}{60} \end{align*} Hence $J = \dfrac{e^{tR/L}(RL\sin 60t - 60L^2\cos 60t)}{R^2+3600L^2}$ and $(*)$ is equivalent to \begin{multline*} \frac{40e^{tR/L}(R\sin 60t - 60L\cos 60t)}{R^2+3600L^2} = Ie^{tR/L} - C\\ \iff I = \frac{40R\sin 60t - 2400L\cos 60t}{R^2+3600L^2} + \frac{C}{e^{tR/L}}\\ \iff I = \frac{\sin 60t - 3\cos 60t}{5} + \frac{C}{e^{t/20}} \end{multline*} Since $I = 1$ at $t = 0$, \[1 = \frac{\sin 0 - 3\cos 0}{5} + \frac{C}{e^0} \iff C = \frac 8 5\] and thus $I = \dfrac{\sin 60t - 3\cos 60t}{5} + \dfrac{8}{5}\exp\dfrac{-t}{20}$. At $t = 0.1$, $I = (\sin 6 - 3\cos 6)/5 + 1.6e^{-1/200} \approx 2.11$ A. \end{document}